LSAT and Law School Admissions Forum

Get expert LSAT preparation and law school admissions advice from PowerScore Test Preparation.

User avatar
 sdb606
  • Posts: 78
  • Joined: Feb 22, 2021
|
#87272
Why is A wrong? I realize it has extreme language but this is a C&E question and I thought the LSAT treats each effect as only having a single corresponding cause. So if herniated discs cause back pain, absence of herniated discs cause absence of back pain. Therefore, 4/5 people can be sure they won't develop back pain.

Of course this is unrealistic in the real world but the LSAT is not the real world.
User avatar
 sdb606
  • Posts: 78
  • Joined: Feb 22, 2021
|
#87273
Actually, I retract my post. I just realized the first sentence says back pain is usually caused by herniated discs which means there could be other causes of back pain so A is not supported.

I assume "usually" cannot also mean "always" in contrast with the way "some"/"most" can equal "all".

Get the most out of your LSAT Prep Plus subscription.

Analyze and track your performance with our Testing and Analytics Package.